LSAT and Law School Admissions Forum

Get expert LSAT preparation and law school admissions advice from PowerScore Test Preparation.

 Administrator
PowerScore Staff
  • PowerScore Staff
  • Posts: 8917
  • Joined: Feb 02, 2011
|
#27035
Complete Question Explanation

Assumption. The correct answer choice is (D)

To make a causal argument based on a study or a survey as this stimulus does (concluding that fatty acids found in fish oil cause a lower incidence of heart disease because people in the study who ate fish often are less likely to develop heart disease), an author must assume that the study itself is valid. In this case, the author must assume that the consumption of fish by the people studied was the cause of the lowered incidence of heart disease, and that some other behavior is not the cause.

Answer choice (A): Vegetarianism is never discussed in the stimulus so it is not a necessary component of the argument.

Answer choice (B): This answer choice confuses a number of test takers, but remember that the author concludes that heart disease is simply related to the amount of fish consumed. So whether or not the fish eaters in the study avoided other dietary elements that would increase their chances of heart disease is irrelevant.

Answer choice (C): Again, red meat is not discussed as being related to heart disease in the stimulus, so it cannot be though to be a necessary piece of the author’s argument.

Answer choice (D): This is the correct answer choice. This answer eliminates a competing potential cause of why the fish eating test subjects have a lower incidence of heart disease: engaging in activities known to augment cardiovascular health.

Answer choice (E): We cannot know that there is a connection between sedentary occupations and heart disease based solely on the stimulus information, so this answer choice is not necessary for the author’s argument to be correct.
 Basia W
  • Posts: 108
  • Joined: Jun 19, 2014
|
#15289
I have a question for page 5-5 question 9.The answer key states that in order to fill in the blank we must show that the fish caused the reduction in heart disease and not something else and it writes off B saying that it is irrelevant. Wouldn't an alternate diet be a possible "other" cause?

Best,

Basia
 Robert Carroll
PowerScore Staff
  • PowerScore Staff
  • Posts: 1787
  • Joined: Dec 06, 2013
|
#15291
Basia,

It is indeed true that for the author to be correct that eating fish helped reduce heart disease, the author had to assume that nothing else was materially different about the middle-aged people in the study. Specifically, if those middle-aged people had done something else that would be likely to have a positive effect on their heart health, then the conclusion that is was the fish would be suspect. In short, the author must assume that this possible contrary evidence does not exist. The assumption here is a Defender; it is an assumption that a possible line of attack doesn't exist.

Answer choice (B) is not relevant because, in that answer, the diet assumed not to exist is one conducive to heart disease. In other words, (B) assumes that those eating fish in the study didn't have an otherwise bad, harmful diet. But if they otherwise had a bad, harmful diet, but also ate fish twice a week, and they had less incidence of heart disease, this certainly would not make me think the bad diet was responsible for less heart disease! (B) cuts the opposite way from the Defender you're looking for - the author must assume that the people in the study aren't doing other good things for their health (besides eating fish). (B) would be an assumption that they aren't doing bad things for their health. The author certainly does not need to assume that; in fact, if you negate (B), it actually makes the conclusion seem stronger - eating fish twice a week was able to improve heart health even with an otherwise heart-unhealthy diet! So the Assumption Negation technique should show why answer choice (B) is in fact opposite to what you want.

Robert Carroll
 rameday
  • Posts: 94
  • Joined: May 07, 2014
|
#15529
So I got this question wrong. But I am a big confused as to why D is right and E is wrong.

For E it says that we cannot know that their is a connection with sedentary occupations and HD based solely on the info in the stimulus. But can't we say the same thing for D that based solely on the info in stimulus we can't know that their is a connection between cardiovascular health and reducing HD?

I ended up choosing B for some really odd reason.

A
 Steve Stein
PowerScore Staff
  • PowerScore Staff
  • Posts: 1153
  • Joined: Apr 11, 2011
|
#15545
Hi A,

In that one, the author says that the scientists' assertion==that omega-3 in fish would provide significant health benefits--got support from the recent study in which people who ate fish at least twice per week had a big reduction in heart disease.

The question asks for the assumption required by the argument.

Correct answer choice (D) provides that the people from the recent study did not do other very healthy activities.

If this had not been the case--if, for example, all of the people involved in the study were also olympians--that would really call into question the assertion that the health benefits resulted from the fish consumption, because this choice presents an alternative cause.

As for answer choice (E), if the subjects had been more likely to lead sedentary lifestyles, that would have strengthened the argument--but the assumption that they did not is not required by the author's argument.

I hope that's helpful--please let me know whether this is clear--thanks!

Steve
 rameday
  • Posts: 94
  • Joined: May 07, 2014
|
#15572
Hello,

Ok I see why D is correct. I think I was on the right track because my pre phrase was that the author assumes that the sample is representative. I chose B so for B to be correct it would have had to say that they did have diet conducive to HD.

So D pretty much does what B should have done.

But I think what I struggle with is this notion of requirement. I still don't quite understand how the fact that the the subjects were not likely to lead sedentary lifestyles is not required by the author's argument.

Like i understand why D is required but I don't understand why E isn't.

Wait nvm, if E had said that the test subjects who ate fish twice a week were more likely that those who did no to lead sedentary lives. So both B and E share the same problem. Would that have made it correct?

A
 Steve Stein
PowerScore Staff
  • PowerScore Staff
  • Posts: 1153
  • Joined: Apr 11, 2011
|
#16131
Hi,

Thanks for your response; if the subjects had been more likely to lead sedentary lifestyles, that would have strengthened the argument--but the assumption that they did not is not required by the author's argument.

I hope that's helpful--please let me know whether this is clear--thanks!

Steve
 Blueballoon5%
  • Posts: 156
  • Joined: Jul 13, 2015
|
#19515
I do not understand how answer B could be wrong and answer D is correct.

In the answer key online, it states that D is correct because it eliminates another cause. However, couldn't we say the same for answer B? I think that a different diet (such as red wine) could be another cause as well.

However, for answer B, the book states that "This answer choice confuses a number of test takers, but remember that the author concludes that heart disease is simply related to the amount of fish consumed. So whether or not the fish eaters in the study avoided other dietary elements that would increase their chances of heart disease is irrelevant."

This answer explanation seems to negate answer D though. We could say that whether or not engaging in regular activities known to augment cardiorespiratory health is irrelevant.

Moreover, I did the negation test for answer B and it seems to attack the argument.

I hope you can help!
 Clay Cooper
PowerScore Staff
  • PowerScore Staff
  • Posts: 241
  • Joined: Jul 03, 2015
|
#19525
Hi BlueBalloon,

I think the key here is the negation technique. Answer choice B, when negated, would read "The test subjects in the recent study who ate fish twice a week DID have a diet that was otherwise conducive to the development of heart disease." If that were true, though, it would make it even more astonishing that the test subjects were nearly 30% less likely to develop heart disease. In other words the negated form of answer choice B is not an attack on the argument, but in fact supports it strongly. Answer choice B, therefore, is incorrect.

Does that help?
 Lawyered
  • Posts: 23
  • Joined: Jun 13, 2017
|
#36130
How do we know this isn't a necessary assumption? Owing to the "assumption required by the argument" I hypothesized it would be a question whereby the answer 100% justifies the stimulus. Gives it validity??

Though I'll be honest the answers didn't really make sense to me. I ended up ruling out D & E but B & C just had me lost... obviously, D was the right answer.

I am still not sure about D primarily because the wording seems confusing. It seems like they are saying the people who ate the fish twice a week were not going to workout more? I eliminated it because it seemed like going off on a tangent discussing irrelevant things sorta like the meat argument. I picked B because I thought it would eliminate an alternate theory (the reasoning used to pick D as the right answer).
Administrator wrote:Complete Question Explanation

Assumption. The correct answer choice is (D)

To make a causal argument based on a study or a survey as this stimulus does (concluding that fatty acids found in fish oil cause a lower incidence of heart disease because people in the study who ate fish often are less likely to develop heart disease), an author must assume that the study itself is valid. In this case, the author must assume that the consumption of fish by the people studied was the cause of the lowered incidence of heart disease and that some other behavior is not the cause.

Answer choice (A): Vegetarianism is never discussed in the stimulus so it is not a necessary component of the argument.

Answer choice (B): This answer choice confuses a number of test takers, but remember that the author concludes that heart disease is simply related to the amount of fish consumed. So whether or not the fish eaters in the study avoided other dietary elements that would increase their chances of heart disease is irrelevant.

Answer choice (C): Again, red meat is not discussed as being related to heart disease in the stimulus, so it cannot be though to be a necessary piece of the author’s argument.

Answer choice (D): This is the correct answer choice. This answer eliminates a competing potential cause of why the fish eating test subjects have a lower incidence of heart disease: engaging in activities known to augment cardiovascular health.

Answer choice (E): We cannot know that there is a connection between sedentary occupations and heart disease based solely on the stimulus information, so this answer choice is not necessary for the author’s argument to be correct.

Get the most out of your LSAT Prep Plus subscription.

Analyze and track your performance with our Testing and Analytics Package.